Search found 51 matches


Great!

Thats a very important inferences you have drawn!

Many thanks!

by nishant1309

Tue Aug 23, 2011 5:17 am
Forum: Research MBA Programs
Topic: Mid 80% Range
Replies: 6
Views: 16263

Thanks Frank!

For a range of 670 - 760 at ISB is to wide a range to really guess how much score i should target.

Well, i beliive the mid value 700-720 would be fair enough in this case.

Thanks!

by nishant1309

Sun Aug 21, 2011 5:00 am
Forum: Research MBA Programs
Topic: Mid 80% Range
Replies: 6
Views: 16263

Dear Frank, Thanks for response! Its not about my score (I've not taken yet!). Its a term that i usually find on school websites or bloomberg website, where they present the key stats of a school. I am not able to comprehend what actually it indicates. For example, refer this link on ISB website: ht...

by nishant1309

Sun Aug 21, 2011 12:27 am
Forum: Research MBA Programs
Topic: Mid 80% Range
Replies: 6
Views: 16263

Mid 80% Range

Dear Folks,

Can anyboby please explain the meaning and interpretation of "Mid 80% (percentile) range" when talking about GMAT score or Salary as declared by B-Schools in their websites?

What does it actually indicates?

by nishant1309

Sat Aug 20, 2011 10:44 pm
Forum: Research MBA Programs
Topic: Mid 80% Range
Replies: 6
Views: 16263

There was a toss between D & E. Eleminated D, because of the redundant "which", because..."three wodded spears..." is an important information and should not be modified by the conjuction "which". Which is genrally used for non-importnat information. Is that a right...

by nishant1309

Mon Apr 25, 2011 12:02 pm
Forum: Sentence Correction
Topic: Stone Age
Replies: 44
Views: 26782

Please note the bold key words in my explanation. Hope it helps. Q: It can be inferred from the passage that a yeast cell with toxic levels of alpha-synuclein will die because We need to look for the ultimate reason: root cause right? Ref. The excerpt from passage and bold key words: By examining th...

by nishant1309

Tue Mar 15, 2011 10:45 pm
Forum: Reading Comprehension
Topic: Parkinsons Disease
Replies: 17
Views: 16175

let us call the most exp. item A, the other two items B and C. We need to find if (0.8A+0.1B+0.1C)/(A+B+C)>0.85 st(1) A=50 and B=C=20 --> (40+18+18)/90<0.85 hence we can answer No on this question, Sufficient; st(2) we have only C or B=15 BUT A can be anything, hence Not Sufficient answer A Henry p...

by nishant1309

Sun Mar 13, 2011 1:56 pm
Forum: Data Sufficiency
Topic: DS Prob from GMAT Prep
Replies: 5
Views: 1524

DS Prob from GMAT Prep

Henry purchased 3 items during a sale. He received 20% discount off regular price of the most expensive item and a 10% discount off regular price off each of the other 2 items. Was the total amount of discount greater than 15% of the sum of the regular prices of all the 3 items? 1. Regular price for...

by nishant1309

Sun Mar 13, 2011 12:58 pm
Forum: Data Sufficiency
Topic: DS Prob from GMAT Prep
Replies: 5
Views: 1524

I concur with C. The conclusion of this argument is vague, but it appears that the author is telling us not to criticize the system for inadequate facilities. His reason - we've spent tons of money improving them. C says - "yeah, you've spent a ton of money, but it's because the system was a m...

by nishant1309

Sun Mar 13, 2011 12:34 pm
Forum: Critical Reasoning
Topic: CR - Weakening the Argument
Replies: 6
Views: 2259

AIM GMAT wrote:OMG !!!!


Anyways whats the source ? Is it any official question ?
It is from one of the mocks i took in my institute....

by nishant1309

Sun Mar 13, 2011 9:00 am
Forum: Critical Reasoning
Topic: CR - Weakening the argument
Replies: 11
Views: 3571

CR - Weakening the Argument

Representative: Our prisons have been criticized for having what some say are inadequate facilities. While it is true that there are still many improvements to be made, critics should consider that improving the prisons has been one of the state's primary concerns in the past few years. In fact, we ...

by nishant1309

Sun Mar 13, 2011 7:00 am
Forum: Critical Reasoning
Topic: CR - Weakening the Argument
Replies: 6
Views: 2259

A. Many businesses experienced significant losses last quarter due to the recession. -- CORRECT , as it talks and confirms recession. B. The last financial quarter ended more than two weeks ago. -- Irrelevant C. Other businesses have experienced similar legal trouble without it affecting their fina...

by nishant1309

Sat Mar 12, 2011 11:40 pm
Forum: Critical Reasoning
Topic: CR - Weakening the argument
Replies: 11
Views: 3571

Hi Nishi, Kindly see my above reply's comments.. I have modified option C. Actually I was looking for an option which helps my initial opinion but none of them match... A is best of the worse option which i chose... So what is OA and OEs @AIM GMAT & HSPA: Thanks for your explanation. I think in...

by nishant1309

Sat Mar 12, 2011 7:39 am
Forum: Critical Reasoning
Topic: CR-Weakening Problem
Replies: 8
Views: 2445

A. Many businesses experienced significant losses last quarter due to the recession. -- CORRECT , as it talks and confirms recession. B. The last financial quarter ended more than two weeks ago. -- Irrelevant C. Other businesses have experienced similar legal trouble without it affecting their fina...

by nishant1309

Sat Mar 12, 2011 7:33 am
Forum: Critical Reasoning
Topic: CR - Weakening the argument
Replies: 11
Views: 3571

HSPA wrote:Hi Nishi..

We are not left with any option other than A... all are out of scope or strengthen
@HSPA: I would be thankful, if you could elaborate your reasoning.

by nishant1309

Sat Mar 12, 2011 12:55 am
Forum: Critical Reasoning
Topic: CR-Weakening Problem
Replies: 8
Views: 2445